Last visit was: 25 Apr 2024, 09:47 It is currently 25 Apr 2024, 09:47

Close
GMAT Club Daily Prep
Thank you for using the timer - this advanced tool can estimate your performance and suggest more practice questions. We have subscribed you to Daily Prep Questions via email.

Customized
for You

we will pick new questions that match your level based on your Timer History

Track
Your Progress

every week, we’ll send you an estimated GMAT score based on your performance

Practice
Pays

we will pick new questions that match your level based on your Timer History
Not interested in getting valuable practice questions and articles delivered to your email? No problem, unsubscribe here.
Close
Request Expert Reply
Confirm Cancel
SORT BY:
Date
Tags:
Show Tags
Hide Tags
User avatar
Intern
Intern
Joined: 28 Aug 2009
Posts: 22
Own Kudos [?]: 90 [60]
Given Kudos: 0
Send PM
Most Helpful Reply
Tutor
Joined: 16 Oct 2010
Posts: 14822
Own Kudos [?]: 64916 [13]
Given Kudos: 426
Location: Pune, India
Send PM
User avatar
Manager
Manager
Joined: 11 May 2010
Posts: 109
Own Kudos [?]: 548 [10]
Given Kudos: 11
Send PM
General Discussion
User avatar
Director
Director
Joined: 25 Aug 2009
Affiliations: Consortium (CGSM.org), NSHMBA
Posts: 929
Own Kudos [?]: 249 [7]
Given Kudos: 113
Location: New Haven
Schools:Yale SOM Class of 2012
 Q48  V51
WE 1: Investment Banking Summer Associate (Boutique tech M&A)
Send PM
Because of the recent transformation of the market. Quore, Inc., must [#permalink]
5
Kudos
2
Bookmarks
talk2pk wrote:
Because of the recent transformation of the market. Quore, Inc., must increase productivity, 10 percent over the course of the next two years, or it will certainly go bankrupt. In fact, however, Quore’s production structure is such that if a 10 percent productivity increase is possible, then a 20 percent increase is attainable.
If the statements above are true, which one of the following must on the basis of them also be true?

(A) It is only Quore’s production structure that makes it possible for Quore to survive the transformation of the market.
(B) Quore will not go bankrupt if it achieves a productivity increase of 20 percent over the next two years.
(C) If the market had not been transformed, Quore would have required no productivity increase in order to avoid bankruptcy.
(D) Because of the transformation of the market, Quore will achieve a productivity increase of 10 percent over the next two years.
(E) If a 20 percent productivity increase is unattainable for Quore, then it must go bankrupt.


Please explain ur answers.


The best way to do this one is to diagram everything you're given in the problem

They tell you that if it doesn't increase by 10%, it will go bankrupt. this is diagrammed as:

(1) if (not)10% -> then bankrupt.

next it says if 10% is possible, then 20% is possible. this is diagrammed as:

10% possible -> then 20% possible

next, you should always look at the converse of statements, as that will usually be the key to unlocking the problem. the converse of this statement is:

(2) if 20% not possible -> 10% not possible

now you have everything you need. if you add equation 1 to the end of equation 2, you will see that E is true:

if 20% not possible ->10 % not possible -> then bankrupt

therefore, E is the answer.
User avatar
Manager
Manager
Joined: 25 Feb 2010
Posts: 208
Own Kudos [?]: 320 [0]
Given Kudos: 10
Send PM
Re: Because of the recent transformation of the market. Quore, Inc., must [#permalink]
I myself choose B,

but after reading your explanation; If I would have thought in reverse direction then I would have chosen E;

In b/w B and E. I'll go with B
because both of these stmts contradict each other.

I think bad question ...

Source please?
User avatar
Manager
Manager
Joined: 06 Aug 2010
Posts: 123
Own Kudos [?]: 695 [2]
Given Kudos: 5
Location: Boston
 Q50  V42
Send PM
Re: Because of the recent transformation of the market. Quore, Inc., must [#permalink]
1
Kudos
1
Bookmarks
It's (E) by contrapositive. You have the implication "If a 10% increase is possible, then a 20% increase is possible." The contrapositive of that is "If a 20% increase isn't possible, then a 10% increase isn't possible." And we know as a given fact that if a 10% increase isn't possible, then the company is going bankrupt. Hence, (E).

(B) is wrong because it's an inverse relationship, which is not equivalent. A -> B does not mean that ~A -> ~B. Nowhere in the original statement does it say that the company WILL DEFINITELY NOT GO BANKRUPT if they improve 10%, or 20%, or even 150%. You just know that they WILL definitely go bankrupt if they don't improve 10%.
User avatar
Manager
Manager
Joined: 17 Nov 2009
Posts: 204
Own Kudos [?]: 262 [2]
Given Kudos: 17
Send PM
Re: Because of the recent transformation of the market. Quore, Inc., must [#permalink]
2
Kudos
rohitgoel15 wrote:
Because of the recent transformation of the market. Quore, Inc., must increase productivity, 10 percent over the course of the next two years, or it will certainly go bankrupt. In fact, however, Quore’s production structure is such that if a 10 percent productivity increase is possible, then a 20 percent increase is attainable.
If the statements above are true, which one of the following must on the basis of them also be true?
It is important to note in the passage that if 20 % is attainable then 10% if easily attainable. Passage quote -"Quore’s production structure is such that if a 10 percent productivity increase is possible, then a 20 percent increase is attainable. But is 10% cannot be attained then 20% cannot be attained too.
(A) It is only Quore’s production structure that makes it possible for Quore to survive the transformation of the market. Nothing given to support this
(B) Quore will not go bankrupt if it achieves a productivity increase of 20 percent over the next two years. Just a restatement of the passage information. No inference
(C) If the market had not been transformed, Quore would have required no productivity increase in order to avoid bankruptcy.This has nothing to do with the question asked
(D) Because of the transformation of the market, Quore will achieve a productivity increase of 10 percent over the next two years. No data supports this prediction.
(E) If a 20 percent productivity increase is unattainable for Quore, then it must go bankrupt.
the passage says Quore’s production structure is such that if a 10 percent productivity increase is possible, then a 20 percent increase is attainable. so if I cannot get 20 that means I cannot get 10 simply because if I get 10 I can get 20.
The explanation given in one of the gmat thread's is:
they tell you that if it doesn't increase by 10%, it will go bankrupt. this is diagrammed as:
(1) if (not)10% -> then bankrupt.
next it says if 10% is possible, then 20% is possible. this is diagrammed as:
10% possible -> then 20% possible
next, you should always look at the converse of statements, as that will usually be the key to unlocking the problem. the converse of this statement is:

(2) if 20% not possible -> 10% not possible
now you have everything you need. if you add equation 1 to the end of equation 2, you will see that E is true:
if 20% not possible ->10 % not possible -> then bankrupt
therefore, E is the answer

But how can we deduce Point 2? if 20% is not possible then 10% may/ may not be possible !


E is relevant and survives POE
User avatar
Intern
Intern
Joined: 31 Mar 2013
Posts: 37
Own Kudos [?]: 170 [0]
Given Kudos: 14
Concentration: General Management, Strategy
Send PM
Re: Because of the recent transformation of the market. Quore, Inc., must [#permalink]
can someone please explain why C is wrong?
User avatar
Retired Moderator
Joined: 16 Jun 2012
Posts: 871
Own Kudos [?]: 8554 [4]
Given Kudos: 123
Location: United States
Send PM
Re: Because of the recent transformation of the market. Quore, Inc., must [#permalink]
4
Kudos
veenu08 wrote:
can someone please explain why C is wrong?


Hi veenu08.

C has a very common logical fallacy.

The logical fallacy is:
Fact: X happens ==> Y MUST happen
Conclusion: X does not happen ==> Y MUST NOT happen

This is wrong conclusion. Why?
Because X is only one of many factors that cause Y happen. If X does not happen, other factor may cause Y happen.

For example:
Fact: I have a flu ==> I MUST stay at home to avoid further health problem
Conclusion: I don't have a flu ==> I MUST NOT stay at home to avoid further health problem

Is the conclusion correct? Nope, because flu is only one of many factors that causes health problem. Even I don't have a flu, but doctor recommends me that "I MUST stay at home" because there's a seasonal flu and many people are infected.

Back to the question.
market transformation happens ==> Quore MUST increase productivity to avoid bankruptcy
market transformation DOES NOT happen ==> Quore MUST NOT increase productivity to avoid bankruptcy
==> WRONG because market transformation IS NOT THE ONLY FACTOR. For example: Quore did not increase productivity, but the company still went to bankruptcy because of aggressive competition or its customer went to bankruptcy and did not make payment on time, blah blah......

Hope it helps.
User avatar
Senior Manager
Senior Manager
Joined: 07 Aug 2011
Posts: 425
Own Kudos [?]: 1751 [1]
Given Kudos: 75
Concentration: International Business, Technology
GMAT 1: 630 Q49 V27
Send PM
Re: Because of the recent transformation of the market. Quore, Inc., must [#permalink]
1
Kudos
souvik101990 wrote:
Because of the recent transformation of the market, Quore Inc. must increase productivity 10 percent over the course of the next two years or it will certainly go bankrupt. In fact, however, Quore's production structure is such that if a 10 percent productivity increase is possible, then a 20 percent increase is attainable. If the statements above are true, which one of the following must on the basis of them also be true?

(A) It is only Quore's production structure that makes it possible for Quore to survive the transformation of the market.

(B) Quore will not go bankrupt if it achieves a productivity increase of 20 percent over the next two years.

(C) If the market had not been transformed, Quore would have required no productivity increase in order to avoid bankruptcy.

(D) Because of the transformation of the market Quore will achieve a productivity increase of 10 percent over the next two years.

(E) If a 20 percent productivity increase is unattainable for Quore, then it must go bankrupt.



could not understand how option E is correct . E is same as saying that ' if i can lift 50kgs, then i can also lift 100kgs, but if i cannot lift 100kgs, then i cannot lift 50 kgs too .'
can someone please explain.

Originally posted by Lucky2783 on 06 Apr 2015, 01:26.
Last edited by Lucky2783 on 06 Apr 2015, 07:03, edited 1 time in total.
User avatar
Intern
Intern
Joined: 24 Mar 2013
Posts: 12
Own Kudos [?]: 17 [2]
Given Kudos: 0
Send PM
Re: Because of the recent transformation of the market. Quore, Inc., must [#permalink]
2
Kudos
It is "must also be true" qq. So, we have to actually find an option which is nothing but echoing the conclusion of the passage.


(A) It is only Quore's production structure that makes it possible for Quore to survive the transformation of the market.
- Nowhere anything similar mentioned in the passage.

(B) Quore will not go bankrupt if it achieves a productivity increase of 20 percent over the next two years.
- It means if quore achieves a productivity of 10 percent then it will go bankrupt. Hence this statement too is not the correct one.

(C) If the market had not been transformed, Quore would have required no productivity increase in order to avoid bankruptcy.
- Market need not to take any action for quora. Irrelevant.

(D) Because of the transformation of the market Quore will achieve a productivity increase of 10 percent over the next two years.
- Nowhere in the passage, mentioned anything similar to this.

(E) If a 20 percent productivity increase is unattainable for Quore, then it must go bankrupt.
- 20 percent productivity will be attainable by itself as soon as Quore attains 10 percent and being unattainable for 10 percent means Quore will go bankrupt. Hence correct.


Experts can also add in case I am missing/misinterpreting something important.
avatar
Manager
Manager
Joined: 04 Jan 2014
Posts: 55
Own Kudos [?]: 56 [1]
Given Kudos: 20
Send PM
Re: Because of the recent transformation of the market. Quore, Inc., must [#permalink]
1
Kudos
Changing market trend--->Need in prod increase of 10%(20% not necessary)--->avoid bankrupt.
If 10% attained 20% is possible.

(A) It is only Quore's production structure that makes it possible for Quore to survive the transformation of the market.
No Info of only structure is reason.. Out

(B) Quore will not go bankrupt if it achieves a productivity increase of 20 percent over the next two years.
Cant say that. 10% will evade bankruption. But 20% need not because no info abt it. What if excess prod leads to bankruption?

(C) If the market had not been transformed, Quore would have required no productivity increase in order to avoid bankruptcy.
Yes. Bankruption is a result of <10% increase in production need that is inturn a result of market transformation. just like no demand, no need..

(D) Because of the transformation of the market Quore will achieve a productivity increase of 10 percent over the next two years.
Not given in passage..

(E) If a 20 percent productivity increase is unattainable for Quore, then it must go bankrupt.
We cant say that. Premise says that if 10% is possible 20% is attainable. that means if 10% is possible WITH EXTRA EFFORTS 20% is possible(means attainable).. But I dont know how this became an answer.. Please help...
User avatar
Senior Manager
Senior Manager
Joined: 07 Aug 2011
Posts: 425
Own Kudos [?]: 1751 [1]
Given Kudos: 75
Concentration: International Business, Technology
GMAT 1: 630 Q49 V27
Send PM
Re: Because of the recent transformation of the market. Quore, Inc., must [#permalink]
1
Kudos
I guess since it is must be true question , we hv to use no information outside the stem And assume all that given is 100% true . Be it 'sun rise from south or dog hunt the sparrow in sky' .
with that in mind , if 20% is unattainable then we shld ask that why it is unattainable? .as per stem 20% will be unattainable only when 10% is not achieved, implying bankruptcy.

Thats the only reasoning i could think of.
Alum
Joined: 19 Mar 2012
Posts: 4341
Own Kudos [?]: 51450 [0]
Given Kudos: 2326
Location: United States (WA)
Concentration: Leadership, General Management
Schools: Ross '20 (M)
GMAT 1: 760 Q50 V42
GMAT 2: 740 Q49 V42 (Online)
GMAT 3: 760 Q50 V42 (Online)
GPA: 3.8
WE:Marketing (Non-Profit and Government)
Send PM
Re: Because of the recent transformation of the market. Quore, Inc., must [#permalink]
Expert Reply
Quote:
The logical fallacy is:
Fact: X happens ==> Y MUST happen
Conclusion: X does not happen ==> Y MUST NOT happen
This is wrong conclusion. Why?
Because X is only one of many factors that cause Y happen. If X does not happen, other factor may cause Y happen.

For example:
Fact: I have a flu ==> I MUST stay at home to avoid further health problem
Conclusion: I don't have a flu ==> I MUST NOT stay at home to avoid further health problem

Is the conclusion correct? Nope, because flu is only one of many factors that causes health problem. Even I don't have a flu, but doctor recommends me that "I MUST stay at home" because there's a seasonal flu and many people are infected.

Back to the question.
market transformation happens ==> Quore MUST increase productivity to avoid bankruptcy
market transformation DOES NOT happen ==> Quore MUST NOT increase productivity to avoid bankruptcy
==> WRONG because market transformation IS NOT THE ONLY FACTOR. For example: Quore did not increase productivity, but the company still went to bankruptcy because of aggressive competition or its customer went to bankruptcy and did not make payment on time, blah blah......

Hope it helps.
Board of Directors
Joined: 11 Jun 2011
Status:QA & VA Forum Moderator
Posts: 6072
Own Kudos [?]: 4689 [1]
Given Kudos: 463
Location: India
GPA: 3.5
WE:Business Development (Commercial Banking)
Send PM
Re: Because of the recent transformation of the market. Quore, Inc., must [#permalink]
1
Kudos
sananoor wrote:
Because of the recent transformation of the market. Quore, Inc., must increase productivity, 10 percent over the course of the next two years, or it will certainly go bankrupt. In fact, however, Quore’s production structure is such that if a 10 percent productivity increase is possible, then a 20 percent increase is attainable.

If the statements above are true, which one of the following must on the basis of them also be true?

(A) It is only Quore’s production structure that makes it possible for Quore to survive the transformation of the market.
(B) Quore will not go bankrupt if it achieves a productivity increase of 20 percent over the next two years.
(C) If the market had not been transformed, Quore would have required no productivity increase in order to avoid bankruptcy.
(D) Because of the transformation of the market, Quore will achieve a productivity increase of 10 percent over the next two years.
(E) If a 20 percent productivity increase is unattainable for Quore, then it must go bankrupt.


Good one !!

If 20% production is possible ====> 10% production is possible

If 20% production is not possible ====> 10% production is not possible====> Bankruptcy.

None but option (E) correctly matches our understanding, hence this is the correct answer.
avatar
Intern
Intern
Joined: 27 Jan 2016
Posts: 1
Own Kudos [?]: [0]
Given Kudos: 0
Send PM
Re: Because of the recent transformation of the market. Quore, Inc., must [#permalink]
I dont see why C) is incorrect. The question states that because of market transformation, Quora must increase productivity.
It doesnt state any other reason why productivity had to be increased, so we cant assume that Quora had to increase productivity otherwise.

I understand why E) is also a possible answer, but I think C) works too.
Board of Directors
Joined: 11 Jun 2011
Status:QA & VA Forum Moderator
Posts: 6072
Own Kudos [?]: 4689 [1]
Given Kudos: 463
Location: India
GPA: 3.5
WE:Business Development (Commercial Banking)
Send PM
Re: Because of the recent transformation of the market. Quore, Inc., must [#permalink]
1
Bookmarks
GallivantingPanda wrote:
I dont see why C) is incorrect. The question states that because of market transformation, Quora must increase productivity.
It doesnt state any other reason why productivity had to be increased, so we cant assume that Quora had to increase productivity otherwise.

I understand why E) is also a possible answer, but I think C) works too.


The stimulus states -

Quote:
Because of the recent transformation of the market.


Market transformation is going to happen and it is beyond the scope of Quore, Inc. to control , thus Market transformation id a external factor which is going to happen the next possible course of action to survive in the changing market place is to increase production by 10% in the following 2 years.

Please feel free to revert in case of any doubt.
avatar
Intern
Intern
Joined: 08 Feb 2016
Posts: 16
Own Kudos [?]: 59 [0]
Given Kudos: 8
Location: Italy
GMAT 1: 730 Q47 V44
Send PM
Re: Because of the recent transformation of the market. Quore, Inc., must [#permalink]
(A) It is only Quore’s production structure that makes it possible for Quore to survive the transformation of the market. We don't know that, the text tells us only that the production structure is such that if 10% increase is attainable then also it's 20%
(B) Quore will not go bankrupt if it achieves a productivity increase of 20 percent over the next two years. We don't know how the market is gonna be. What if another change in the market happens the year after and Quore needs a 50% increase?
(C) If the market had not been transformed, Quore would have required no productivity increase in order to avoid bankruptcy. The text doesn't say anything about the previous situation. Quore could have needed an increase of 5%. It's not the 10% required after the market transformation but it's still an increase.
(D) Because of the transformation of the market, Quore will achieve a productivity increase of 10 percent over the next two years. Not true. Quore needs to reach a 10% increase or it will go bankrupt for sure, but we don't know if Quore will succeed.
(E) If a 20 percent productivity increase is unattainable for Quore, then it must go bankrupt. True. If 20% is possible then 10% is possible, and if 10% is not reached Quore will go bankrupt
Manager
Manager
Joined: 11 Jun 2017
Posts: 55
Own Kudos [?]: 101 [0]
Given Kudos: 8
Send PM
Re: Because of the recent transformation of the market. Quore, Inc., must [#permalink]
Because of the recent transformation of the market. Quore, Inc., must increase productivity, 10 percent over the course of the next two years, or it will certainly go bankrupt. In fact, however, Quore’s production structure is such that if a 10 percent productivity increase is possible, then a 20 percent increase is attainable.
If the statements above are true, which one of the following must on the basis of them also be true?

This a must be true question with all premises and no conclusion. The argument states that if Quore Inc does not increase productivity >>it will go bankrupt. Second: if a 10% is possible >> 20% increase is attainable.

Let's look at the answer options one by one.

(A) It is only Quore’s production structure that makes it possible for Quore to survive the transformation of the market: Nowhere in the argument is this stated. It does not mention about other production structures other than Quore's.

B) Quore will not go bankrupt if it achieves a productivity increase of 20 percent over the next two years.- This option mistakes a sufficient condition for a necessary condition. If a 10%>20% increase is not guaranteed it will go bankrupt. However, this does not mean that a 20% increase will necessarily mean no bankruptcy. X leads to Y can meant "Not Y>>Not X". However, it can't mean Not X leads to Not Y. X is "not achieving 10% here" and Y is "bankruptcy".
So, this means not achieveing 10% leads to bankruptcy. However, achieving 10% does not lead to no bankruptcy situation (notX>>notY not being equal )

(C) If the market had not been transformed, Quore would have required no productivity increase in order to avoid bankruptcy. - nothing substantiates this

(D) Because of the transformation of the market, Quore will achieve a productivity increase of 10 percent over the next two years. - not necessarily true, they may or may not achieve

(E) If a 20 percent productivity increase is unattainable for Quore, then it must go bankrupt. - Yes!. This is a reformed version of X>>Y. If it does attain the increase, it will go bankrupt. Since 10%>>20% (as given in the argument)., so if 20% is not there, it means 10% is also not there. Hence, 20% can be used in place 10% here. E is correct.
Intern
Intern
Joined: 14 May 2017
Posts: 41
Own Kudos [?]: 10 [0]
Given Kudos: 325
Location: India
GPA: 3.5
WE:Business Development (Education)
Send PM
Re: Because of the recent transformation of the market. Quore, Inc., must [#permalink]
Good question.

Here is my take.

Let's begin by identifying the necessary conditions and the results.

Necessary conditions
#1 Transformation of the markets
#2 Productivity increase

Sufficient condition ? Not given in the passage

Result
No bankcruptcy

A. No. The production structure is such that a 10% increase in productivity is only possible not guaranteed. Moreover we cannot say that there is a link between survival and productivity. Maybe there is. But we cannot say for sure
D. Not necessary. We are only told that a 10% increase is POSSIBLE



B. Satisfying a NECESSARY CONDITION does not guarantee the RESULT. But satisfying both NECESSARY AND SUFFICIENT CONDITIONS only guarantees the RESULT. In the question we do not have any sufficient conditions mentioned. So either they are met or not met. So we cannot be sure. OUT
C According to this statement if #1 did not happen, #2 would not have happened. Again #1 is a NECESSARY CONDITION and NOT a sufficient condition. There could be other reasons to increase productivity. OUT


E Only this choice logically follows. If 20% is not attainable, 10% is not possible and if 10% is not possible then bankruptcy is imminent.T THE CONTRAPOSITIVE MUST BE TRUE. IF SUFFICIENT conditions are met but NECESSARY CONDITIONS are not met, then the we do not get the RESULT
GMAT Club Bot
Re: Because of the recent transformation of the market. Quore, Inc., must [#permalink]
 1   2   
Moderators:
GMAT Club Verbal Expert
6921 posts
GMAT Club Verbal Expert
238 posts
CR Forum Moderator
832 posts

Powered by phpBB © phpBB Group | Emoji artwork provided by EmojiOne